Vous êtes sur la page 1sur 9

Universidad Tcnica Federico Santa Mara

Departamento de Matemtica
Campus Santiago

Ejercicios de Induccin
1. Demuestre por induccin que 1 + 2 + 3 + . . . + n =

n(n + 1)
, n N.
2

2. Demuestre por induccin que 1 + a + a2 + . . . + an =

3. Demuestre por induccin que 1 + 23 + 33 + . . . + n3 =

1 an+1
, donde a R y a 6= 1.
1a


n(n + 1)
2

2
, n N.

4. Demuestre por induccin que si f (x) = xn , siendo n N entonces f 0 (x) = n xn1 .


5. Demuestre usando induccin, que en un polgono convexo se pueden trazar

n(n3)
2

diagonales.

6. Probar por Induccin que: 10n 9n 1 es mltiplo de 81, n 1.


n
X

7. Demuestre por Induccin: n N,

k5k =

k=1

5 + (4n 1)5n+1
16

8. Muestre por induccin que para todo natural n el nmero 22n 1 es divisible por 3
9. Demuestre por induccin que para todo nmero natural, la expresin 4n3 + 8n es divisible por 12.
10. Sea a1 = 0 y a2 = 1 se define la sucesin an+2 =

an+1 +an
2

11. Demuestre por induccin que si f (x) = ln(x) entonces

df
dx

con n > 0. Entonces an =


=

2
3

h
1


1 n1
2

(1)n1 (n1)!
.
xn

12. Demostrar por induccin que : 834n 2 972n + 1 es divisible por 16.
13. Demostrar que: n N :
14. Demostrar que

Pn

n1+

1
k=1 k(k+1)

1
2

1
3

1
4

+ ... +

1
n

n.

n
n+1 .

15. Demostrar por induccin que :11n+2 + 122n+1 es divisible por 133.
16. Demostrar que si an+1 = 2 an + 1 entonces an = 2n1 (a1 + 1) 1
17. Sabiendo que 4 =

3
a1

3
a2

+ a1 =

3
a3

+ a2 = ... =

3
an+1

+ an demostrar que an =

18. Demostrar que: Si n N : an+1 = (1 + x) an n x y a1 = 0 entonces an =


19. Demostrar que: n N : 2n+4 > (n + 4)2 .
20. Demostrar por induccin que :22n+1 9 n2 + 3 n 2 es divisible por 54.

1
x

3n+1 3
3n+1 1 .

[1 + n x (1 + x)n ] ; x 6= 0.

21. Demostrar que cos() cos(2 ) cos(22 ) ... cos(2n1 ) =

sen(2n )
2n sen() .

22. Demostrar que cos(n) = (1)n .


23. Probar que la suma de los ngulos interiores de un polgono convexo regular de n lados es (n 2) .
24. Sea n un nmero natural y di (n) el dgito i de n y sea l(n) el nmero de dgitos de n. Demostrar que
Pl(n)
n i=1 di (n) es un mltiplo de 9.
25. La secuencia de Fibonacci es a0 = 0 ; a1 = 1 ; an+1 = an + an1 , demuestre que an >


3 n1
.
2

26. Demostrar que: n N : xn y n es divisible por (x y)


Pn

27. Sea p N demuestre que


28. Demuestre que

k=1

Pn

k=1 (1)

k1

k (k + 1) (k + 2) (k + 3) ...(k + p) =

n(n+1)(n+2)...(n+p+1)
.
p+2

k 2 = (n + 1) (2n + 1).

Ejercicios Resueltos
1. Demostrar que n 1,
n
X
1
1
1
3
1
1
= 1 + 2 + 2 + + 2
2
l
2
3
n
2 n+1
l=1

Solucin:
Sea P (n) :

n
X
1
1
1
1
3
1
= 1 + 2 + 2 + + 2
, n N una funcin proposicional.
2
l
2
3
n
2 n+1
l=1

Por demostrar que P (1) es verdadera.


En efecto, se tiene que si n = 1 1

3
2

1
2

= 1.

Si n = k, P (k) :

k
X
1
1
1
1
3
1
= 1 + 2 + 2 + + 2
l2
2
3
k
2 k+1
l=1

Por demostrar que P (n) se cumple para n = k + 1.


En efecto,
1+

1
1
1
1
3
1
1
3
k
+ 2 + + 2 +

+
=
,
22
3
k
(k + 1)2
2 k + 1 (k + 1)2
2 (k + 1)2

pero,
3
k
3
1
k
1

k 2 + 2k k 2 + 2k + 1 0 1.
2 (k + 1)2
2 k+2
(k + 1)2
k+2
Por lo tanto, 1 +

1
22

1
32

+ +

Luego, P (n) se cumple n Z.

1
k2

1
(k+1)2

3
2

1
k+2 .

2. Demostrar que n N,

52n + (1)n+1 es divisible por 13

Solucin:
Sea P (n) : 52n + (1)n+1 = 3l,

lZ

Por demostrar que P (1) es verdadero.


En efecto:
P (1) : 52 + (1)2 = 25 + 1 = 26 que es divisible por 13.
Asumamos vlido para n = k, es decir, P (k) : 52k + (1)k+1 = 13l1 , l1 Z.
Por demostrar que P (k + 1) es verdadero. Es decir, por demostrar:
P (k + 1) : 52(k+1) + (1)k+2 = 3l, l Z
En efecto:
52(k+1) + (1)k+2

=
=
=
=
=
=
=
=

52k 52 + (1)k+2
52k 25 + (1)k+2
52k (26 1) (1)k+1
26 52k 52k (1)k+1
26 52k (52k + (1)k+1
26 52k 13l1
13(2 52k l1 )
13l

Por lo tanto, P (k + 1) es verdadero.


Luego la proposicin P (n) se cumple n N.
3. Probar por Induccin que: 10n 9n 1 es mltiplo de 81, n 1.
Solucin:
Considerar la funcin proposicional p(n) : 10n 9n 1 es mltiplo de 81.
p(1) : 101 9(1) 1 = 10 9 1 = 0 y

0 es mltiplo de 81.

p(1) es Verdadero

Asumamos vlido para n = k, es decir, 10k 9k 1 es mltiplo de 81.


Por demostrar que P (n) se
En efecto,
10k+1 9(k + 1) 1 =
=
=
=
=

cumple para n = k + 1.
10k 10 9k 9 1
10k (9 + 1) 9k 1 9
(10k 9k 1) + 9(10k 1)
(10k 9k 1) + 9(10k 9k 1 + 9k)
(10k 9k 1) + 9(10k 9k 1) + 81k

Esta ltima expresin es mltiplo de 81, pues 10k 9k 1 es mltiplo de 81, por hipotesis de induccin
y 81k es mltiplo de 81.
As 10n 9n 1 es mltiplo de 81, para todo n N.

4. Muestre que para todo n N se cumple


n
X

nx
2

sin
sin (kx) =

sin

x
2

sin

k=1

(n+1)x
2

donde x 6= 2l, l Z. Ind.:


sin

 nx 
2

+ 2 sin

x
2


cos

(n + 1)
x
2


= sin

(n + 2) x
2

Solucin: Si n = 1 se cumple
sin x =

x
2

sin

sin

sin

x

2x
2

(n+1)x
2

supongamos que
n
X

sin
sin (kx) =

nx
2

sin

sin

k=1

x
2

entonces
n+1
X

sin
sin (kx)

k=1

nx
2

sin

(n+1)x
2


+ sin ((n + 1) x)
sin x2




(n+1)x
sin nx
+ sin x2 sin ((n + 1) x)
2 sin
2

sin x2








(n+1)x
(n+1)
sin nx
+ sin x2 2 sin (n+1)
2 sin
2
2 x cos
2 x

sin x2




  
x
sin (n+1)x
2
(n + 1)
nx

+
2
sin
cos
x
sin
2
2
2
sin x2

pero (ver indicacin)



(n + 1)
sin
+ 2 sin
cos
x
2
2
2
 nx 
x 
n 
x
n 
 x 
sin
+ 2 sin
cos
x cos
sin
x sin
2 
2  2
2
 2
 n 2
 nx
n
2 x
sin
+ sin x cos
x 2 sin
sin
x
2 
2  2
 nx
 2x 
n
sin
1 2 sin2
+ sin x cos
x
2 
2  
2
 nx
n
sin
cos x + sin x cos
x
2
2
 nx

sin
+x
2

(n + 2) x
sin
2
 nx 

=
=
=
=
=
=

x

as
n+1
X
k=1

sin
sin (kx) =

(n+1)x
2

sin



sin (n+2)x
2

x
2

por el principio de induccin la frmula es vlida para todo n N.

5. Demuestre que para todo n N el nmero 7n 2n es divisible por 5.


Solucin: Consideremos la funcin proposicional p (n) : 7n 2n es divisible por 5.
p (1) : 7 2 = 5 es divisible por 5 lo cual es verdadero.
Si n = k, entonces p(k) es Verdadero
Es decir, 7k 2k es divisible por 5.
Por demostrar que P (n) se cumple para n = k + 1. Es decir, por demostrar que:
7k+1 2k+1 es divisible por 5
En efecto
7k+1 2k+1



7 7k 2 2k


(5 + 2) 7k 2 2k


5 7k + 2 7k 2k

=
por la hiptesis de induccin, existe un n0 ta que

7k 2k = 5n0
se sigue
7k+1 2k+1


5 7k + 2 (5n0 )

5 7k + 2n0

5v0

con v0 Z entonces 7k+1 2k+1 es divisible por 5.


Por el principio de induccin para todo n N el nmero 7n 2n es divisible por 5.
3

6. Muestre que para todo natural n el nmero (n + 1) n 1 es divisible por 3.


Solucin: Para n = 1 se tiene
23 2 = 6 = 2 3
3

supongamos que (k + 1) k 1 es divisible por 3, esto es, existe u0 Z tal que


3

(k + 1) k 1 = 3u0
3

mostremos que (k + 2) k 2 es divisible por 3, en efecto


3

(k + 2) k 2

(k + 1 + 1) k 1 1
3

(k + 1) + 3 (k + 1) + 3 (k + 1) + 1 k 1 1


3
2
= (k + 1) k 1 + 3 (k + 1) + (k + 1)


2
= 3u0 + 3 (k + 1) + (k + 1)
=

3v0
3

se sigue que es divisible por 3. Por el principio de induccin, para todo natural n el nmero (n + 1) n 1
es divisible por 3.

7. Muestre que para todo natural n el nmero 22n 1 es divisible por 3.


Solucin: Queremos demostrar que 4n 1 es divisible por 3 para todo n N, utilizaremos el principio de
induccin. Para n = 1 se tiene 4 1 = 3 es divisible por 3. Supongamos que 4k 1 es divisible por 3, esto es,
existe u0 Z tal que
4k 1 = 3u0
mostremos que eso implica que 4k+1 1 es divisible por 3, en efecto
4k+1 1

4k 4 1

4k (3 + 1) 1

3 4k + 4 k 1

3 4k + 3u0

3v0

donde v0 = 4k + u0 Z, se sigue, por el principio de induccin que para todo natural n el nmero 22n 1 es
divisible por 3
8. Muestre que para todo natural n el nmero n3 + 5n es divisible por 6.
Solucin: Queremos demostrar que n3 + 5n es divisible por 6 para todo n N, utilizaremos el principio de
induccin. Para n = 1 se tiene 1 + 5 = 6 es divisible por 6. Supongamos que k 3 + 5k es divisible por 6, esto
es, existe u0 Z tal que
k 3 + 5k = 6u0
3

mostremos que eso implica que (k + 1) + 5 (k + 1) es divisible por 6, en efecto


3

(k + 1) + 5 (k + 1)

k 3 + 3k 2 + 8k + 6

=
=

k 3 + 5k + 3k 2 + 3k + 6

6u0 + 3 k 2 + k + 6

6u0 + 3k (k + 1) + 6

el nmero k (k + 1) siempre es par, pues como k y k + 1 son nmeros naturales consecutivos uno de los dos
debe ser par, se sigue
3

(k + 1) + 5 (k + 1)

6u0 + 6p0 + 6

6v0

donde v0 = u0 + p0 + 1 Z, se sigue, por el principio de induccin que para todo natural n el nmero n3 + 5n
es divisible por 6
9. Muestre que para todo natural n el nmero 4n + 15n 1 es divisible por 3.
Solucin: Queremos demostrar que 4n + 15n 1 es divisible por 3 para todo n N, utilizaremos el principio
de induccin. Para n = 1 se tiene 4 + 15 1 = 18 es divisible por 3. Supongamos que 4k + 15k 1 es divisible
por 3, esto es, existe u0 Z tal que
4k + 15k 1 = 3u0
mostremos que eso implica que 4k+1 + 15 (k + 1) 1 es divisible por 3, en efecto
4k+1 + 15 (k + 1) 1

4 4k + 15k 1 + 15

=
=

4k + 15k 1 + 3 4k + 3 5


4k + 15k 1 + 3 4k + 5

3u0 + 3 4k + 5

3v0

donde v0 = u0 + 4k + 5 Z, se sigue, por el principio de induccin que para todo natural n el nmero
4n + 15n 1 es divisible por 3.

10. Demuestre que para cualquier x > 0 y cualquiera que sea el nmero natural n se cumple la desigualdad:
xn + xn2 + xn4 + . . . +

1
1
1
+ n2 + n n + 1.
xn4
x
x

(1)

Solucin:
a) Para n = 1 la desigualdad (1) es
x+

1
2
x

(2)

Como x > 0, (2) es equivalente a mostrar que (x 1)2 0, lo cual es evidente.


b) Para n = 2 la desigualdad (1) es
x2 + 1 +

1
3
x2

(3)

Como (2) se cumple para todo x > 0, sustituyendo x por x2 obtenemos


x2 +

1
2,
x2

luego sumando 1 a la desigualdad anterior se obtiene (3).


c) Supongamos que la desigualdad (1) se cumple para n = k, o sea, que

xk + xk2 + xk4 + . . . +

1
1
1
+ k2 + k k + 1,
xk4
x
x

(4)

donde k es un nmero natural. Se probara que la desigualdad (1) tambien se cumple para n = k + 2, o
sea, que
xk+2 + xk + xk2 + . . . +

1
xk2

1
1
+ k+2 k + 3,
k
x
x

(5)

Como (2) se cumple para todo x > 0, sustituyendo x por xk+2 obtenemos
xk+2 +

1
xk+2

2.

(6)

Luego, sumando las desigualdades (4) y (6), obtenemos la desiguladad (5).


Resumiendo:
En a), b) hemos demostrado la desigualdad (1) para n = 1 y para n = 2.
En c) hemos demostrado que, siendo vlida la desigualdad (1) para n = k, tambien lo es para n = k + 2.
Por lo tanto, de b) y c), si la desigualdad (1) es valida para n = 2k, entonces tambien es valida para
n = 2k + 2. Es decir, que la desigualdad (1) se cumple para cualquier nmero par. De igual manera, de
a) y c), si n = 2k 1, entonces la desigualdad (1) tambien es valida para n = 2k + 1. Es decir, que la
desigualdad (1) se cumple para cualquier nmero impar.
11. Demuestre que, n N:
11
19
n2 + 3n + 1
n (2n + 3)
5
+
+
+

+
=
2
2
2
2
2
2
2
2
1 2
2 3
3 4
n (n + 1)
(n + 1)2

Solucin:

()

Para n = 1:

5
5
15
= =
12 22
4
(1 + 1)2

Luego () es vlido para n = 1.


Supongamos que () es vlido para n, esto es:
12

11
19
n2 + 3n + 1
n (2n + 3)
5
+ 2 2 + 2 2 + + 2
=
2
2
2 3
3 4
n (n + 1)2
(n + 1)2

Para n + 1:
5
11
19
n2 + 3n + 1
(n + 1)2 + 3(n + 1) + 1
+
+
+

+
+
12 22
22 32
32 42
n2 (n + 1)2
(n + 1)2 (n + 2)2


1
n2 + 5n + 5
H.I. n (2n + 3) (n + 1)2 + 3(n + 1) + 1
=
+
=
n (2n + 3) +
(n + 1)2
(n + 1)2 (n + 2)2
(n + 1)2
(n + 2)2
1
=
(n + 1)2

n (2n + 3) (n + 2)2 + n2 + 5n + 5
(n + 2)2

1
=
(n + 1)2

(n + 1)3 (2n + 5)
(n + 2)2

(n + 1) (2(n + 1) + 3)
(n + 1) (2n + 5)
=
(n + 2)2
(n + 2)2

Por lo tanto () es vlido para n + 1 y por Primer Principio de Induccin, es vlido n N,


esto es:
n N

5
11
19
n2 + 3n + 1
n (2n + 3)
+
+
+

+
=
2
2
2
2
2
2
2
2
1 2
2 3
3 4
n (n + 1)
(n + 1)2

12. Usando induccin muestre que


dn x

n/2 x
(e
sin
x)
=
2
e
sin
x
+
n
dxn
4
para n N y x R. Utilizando la frmula anterior y la derivada de un producto calcular
n  

X
n

sin x + k
k
2
k=0

Solucin:
Si n = 1 la frmula es correcta
d x
(e sin x)
dx

= ex (cos x + sin x)


= 21/2 ex sin x +
4

supongamos que

dk x

k/2 x
(e
sin
x)
=
2
e
sin
x
+
k
dxk
4

entonces
dk+1 x
(e sin x)
dxk+1


d  k/2 x

2 e sin x + k
dx 
4




k/2
x
+ ex cos x + k
= 2
e sin x + k
4
4


k/2 1/2 x
= 2 2 e sin x + k +
4
4 


= 2(k+1)/2 ex sin x + (k + 1)
4
=

por el principio de induccin la frmula es vlida para cada n N.


De la frmula para la derivada nsima de un producto se tiene
dn x
(e sin x)
dxn

n  
X
n
k=0
n 
X

(nk)

(ex )

(sin x)

(k)


n x
(k)
e (sin x)
k
k=0
n  

X
n x

=
e sin x + k
2
k
k=0
n  

X
n

sin x + k
= ex
k
2
=

k=0

pero



dn x
n/2 x
(e
sin
x)
=
2
e
sin
x
+
n
dxn
4

as
2

n  

X

n

x
e sin x + n
=e
sin x + k
4
k
2

n/2 x

k=0

de donde obtenemos
2

n/2

n  


 X n
sin x + k
sin x + n
=
k
4
2

k=0

Vous aimerez peut-être aussi